Question and Answers Forum

All Questions      Topic List

Others Questions

Previous in All Question      Next in All Question      

Previous in Others      Next in Others      

Question Number 37728 by kunal1234523 last updated on 17/Jun/18

1+n+((n(n−1))/(2!))+((n(n−1)(n−2))/(3!))+((n(n−1)(n−2)(n−3))/(4!))+........=

$$\mathrm{1}+\mathrm{n}+\frac{\mathrm{n}\left(\mathrm{n}−\mathrm{1}\right)}{\mathrm{2}!}+\frac{\mathrm{n}\left(\mathrm{n}−\mathrm{1}\right)\left(\mathrm{n}−\mathrm{2}\right)}{\mathrm{3}!}+\frac{\mathrm{n}\left(\mathrm{n}−\mathrm{1}\right)\left(\mathrm{n}−\mathrm{2}\right)\left(\mathrm{n}−\mathrm{3}\right)}{\mathrm{4}!}+........= \\ $$

Answered by tanmay.chaudhury50@gmail.com last updated on 17/Jun/18

(1+x)^n =1+nC_1 x+nC_2 x^2 +nC_3 x^3 +...+nC_n x^n   put x=1  2^n =1+nC_1 +nC_2 +nC_3 +...+nC_n   2^n =1+n+((n(n−1))/(2!))+((n(n−1)(n−2))/(3!))  so the answer is 2^n

$$\left(\mathrm{1}+{x}\right)^{{n}} =\mathrm{1}+{nC}_{\mathrm{1}} {x}+{nC}_{\mathrm{2}} {x}^{\mathrm{2}} +{nC}_{\mathrm{3}} {x}^{\mathrm{3}} +...+{nC}_{{n}} {x}^{{n}} \\ $$$${put}\:{x}=\mathrm{1} \\ $$$$\mathrm{2}^{{n}} =\mathrm{1}+{nC}_{\mathrm{1}} +{nC}_{\mathrm{2}} +{nC}_{\mathrm{3}} +...+{nC}_{{n}} \\ $$$$\mathrm{2}^{{n}} =\mathrm{1}+{n}+\frac{{n}\left({n}−\mathrm{1}\right)}{\mathrm{2}!}+\frac{{n}\left({n}−\mathrm{1}\right)\left({n}−\mathrm{2}\right)}{\mathrm{3}!} \\ $$$${so}\:{the}\:{answer}\:{is}\:\mathrm{2}^{{n}} \\ $$

Commented by kunal1234523 last updated on 17/Jun/18

Commented by kunal1234523 last updated on 17/Jun/18

according to wolfram alpha it can not be   determine in general form but you did it  are you a genius

$$\mathrm{according}\:\mathrm{to}\:\mathrm{wolfram}\:\mathrm{alpha}\:\mathrm{it}\:\mathrm{can}\:\mathrm{not}\:\mathrm{be}\: \\ $$$$\mathrm{determine}\:\mathrm{in}\:\mathrm{general}\:\mathrm{form}\:\mathrm{but}\:\mathrm{you}\:\mathrm{did}\:\mathrm{it} \\ $$$$\mathrm{are}\:\mathrm{you}\:\mathrm{a}\:\mathrm{genius} \\ $$

Commented by kunal1234523 last updated on 17/Jun/18

This question is arived to me when I am   working on sets. I came to know that no. of  subsets of  a set is 2^n  where n is no.of terms  or objects in the set. so I tried to prove it and  I end up in the question that I had asked.

$$\mathrm{This}\:\mathrm{question}\:\mathrm{is}\:\mathrm{arived}\:\mathrm{to}\:\mathrm{me}\:\mathrm{when}\:\mathrm{I}\:\mathrm{am}\: \\ $$$$\mathrm{working}\:\mathrm{on}\:\mathrm{sets}.\:\mathrm{I}\:\mathrm{came}\:\mathrm{to}\:\mathrm{know}\:\mathrm{that}\:\mathrm{no}.\:\mathrm{of} \\ $$$$\mathrm{subsets}\:\mathrm{of}\:\:\mathrm{a}\:\mathrm{set}\:\mathrm{is}\:\mathrm{2}^{\mathrm{n}} \:\mathrm{where}\:\mathrm{n}\:\mathrm{is}\:\mathrm{no}.\mathrm{of}\:\mathrm{terms} \\ $$$$\mathrm{or}\:\mathrm{objects}\:\mathrm{in}\:\mathrm{the}\:\mathrm{set}.\:\mathrm{so}\:\mathrm{I}\:\mathrm{tried}\:\mathrm{to}\:\mathrm{prove}\:\mathrm{it}\:\mathrm{and} \\ $$$$\mathrm{I}\:\mathrm{end}\:\mathrm{up}\:\mathrm{in}\:\mathrm{the}\:\mathrm{question}\:\mathrm{that}\:\mathrm{I}\:\mathrm{had}\:\mathrm{asked}.\: \\ $$

Terms of Service

Privacy Policy

Contact: info@tinkutara.com